Download as pdf or txt
Download as pdf or txt
You are on page 1of 22

DAY-21

DAY-26
DAY 21

DAY 01

1
DAY-21

Dear Students,
I hope this message finds you well and motivated for your upcoming Bank Mains Exams. As you
prepare to tackle this significant challenge, I want to offer you some words of encouragement and
guidance to help you succeed.
1. Stay Committed: Remember why you started this journey. Keep that motivation alive and use it
to fuel your dedication towards your goal. The journey might be tough, but your determination
will see you through.
2. Plan Wisely: Create a detailed study plan that covers all the subjects and topics you need to study.
Allocate time wisely, giving more attention to your weaker areas while also revising your strengths.
3. Practice Regularly: Consistent practice is key to success. Rank File is a collection of most effective
and exam related questions. This will help you manage your time effectively during the exam.
4. Stay Healthy: Good physical and mental health is crucial for effective preparation. Eat well,
exercise, and take breaks to recharge. Don't compromise your well-being for the sake of studying.
5. Stay Positive: Maintain a positive attitude, even when facing difficult concepts or a challenging
mock test. A positive mindset will help you overcome obstacles more easily.
6. Time Management: Learn to manage your time during the exam. Don't get stuck on a single
question; move on and return to it later if necessary. Allocate time wisely to different sections.
7. Believe in Yourself: You've put in the effort, and you are capable. Believe in your abilities and stay
confident throughout the exam.
Remember, success is not just about the destination but also about the journey. Embrace the learning
process, and every challenge you face will be a stepping stone towards your goal. Keep your eyes on
the prize, and you will surely excel in your Bank Mains Exam.
Wishing you the best of luck and success!
[SHANTANU SHUKLA]
Sr. Educator, Adda 247

2
DAY-21

DAY 21
S. No. Topic Sub Topic No. of Questions
1 Arithmetic With Variable 6
2 Number series Wrong 3
3 Quantity Comparison 2 statements 3
4 Data Interpretation Caselet (Mix) 4
5 Data Interpretation Caselet (t&w) 3
6 Data Interpretation Iine + Column 4
Column
7 Data Interpretation 4
(Partnership)
8 Data Sufficiency 2 statements 4
9 Algebra 3
10 Data Interpretation Table + Mixture 3
Line + table +
11 Data Interpretation 3
Mixture
Total 40

3
DAY-21
1. Ram can complete a piece of work in ‘a’ day 4. Two boats P and Q covers 240 km and 340 km
by working ‘m’ hours per day, and Shyam can downstream distance in __ hours and __ hours
complete the same work in ‘b’ days by respectively. Upstream speed of boat P is 2
working ‘n’ hours per day. If each of them km/hr less than the upstream speed of boat Q
reduces his working hours per day by 4 whose speed in still water is 24 km/hr. Speed
hours, each of them would take 5 days more of boat P in still water is 120% more than that
to finish the work (m, n < 16). Which of the of stream speed. The respective ratio of
following is the time taken if Ram and Shyam downstream of boat P to the speed of boat Q
started working together such that Ram in still water is __ respectively. Stream is
worked for ‘m’ hours per day and Shyam flowing with speed __ km/hr.
worked for ‘n’ hours per day? (m, n,a, b are all Which of the following option is possible to fill
integers) the given blanks in same order?
I. 5days (a) 10, 17, 3: 2, 12 (b) 7.5, 17, 5: 4, 10
II. 4days (c) 7.5, 10, 4: 3, 10 (d) 10, 10, 4: 3, 12
10 (e) 12, 20, 5: 4, 14
III. 3
days
5. Taju invested Rs. (5E + 1200) in a scheme A
IV. 2.5 days
1 offers compound interest at 25% per annum
V. 3 2 days from two years. Interest earned from this
(a) Both I and V (b) I, III and IV scheme he invested in another scheme B
(c) Both I and III (d) Both III and IV offers compound interest at 20% per annum,
(e) II, IV and V which amounts to Rs. C after two years. After
that he spends Rs. E and invested remaining
Directions(Q2-3) : Read the data carefully and amount (what he received from scheme B) in
answer the following questions. another scheme D in simple interest for five
Below given are three equations numbered I, II, and years at 12% per annum. Simple interest
III. earned by Taju is Rs. (2E + 80). Find the
I: 5m2 - d× 𝑚 + 14 = 0 money invested in scheme offers simple
II: r*n2 – s*n + 2 = 0 interest?
III: e*t2 – 17t + 3s = 0 (a) 12000 (b) 9000
One root of the equation I is 2/5, one non-integer (c) 10000 (d) 16000
root in equations I and II is common. Other root of (e) None of these
equation II is 1 and one integer root in equations I 6. A tank is partially filled up to volume M liters.
and III is common. At the same instant 24 holes generated at the
2. After adding which of the following term in bottom, and an inlet pipe is connected to a
equation III, both the roots of the equation tank, so it takes 8 hours to empty the tank. If
will become equal? 20 holes are generated, then tank is emptied
in 16 hours. If (2N + 8) holes are generated
(a) 121/8 (b) 49/4
then tank is emptied in 4 hours. Find the
(c) 225/8 (d) 169/8
value of N?
(e) 81/8
(a) 6 (b) 8
3. Which of the following is equivalent to the (c) 24 (d) 10
value of d? (e) None of these
(a) r(s + e) (b) rs + e
(c) rs - e (d) r + se Direction(Q7-9): Two quantities (I) and (II) are
(e) (r + s)e given. You have to solve both the equations and
give Answer key:

4
DAY-21
7. Quantity I: Two persons Sam and Tom picking two balls from bag without replacement
together can do a work in 100/9 days and such that one is pink and one is white is 16/105.
ratio of efficiency person Sam to Tom is (1/4): If total balls in bag is A, what is the value of (100A
(1/5). Persons Sam and Tom together work – 20B).
for first 7 days and then Tom is replaced by
(a) Quantity I < Quantity II
another person David. Now, it takes 4(14/15)
(b) Quantity I > Quantity II
more days to complete the remaining work.
What is the total time taken by person David (c) Quantity I = Quantity II
alone to complete the work? (d) Quantity I ≤ Quantity II
Quantity II: Shailesh has Rs.12000 with him out (e) Quantity I ≥ Quantity II
of which he spent 'm%' on shopping and from the 9. Quantity I. In an examination average marks
remaining he spent 'n%' on other activities. Now of B students are A. Later it was found that
he has Rs.2880 remains with him. If the marks of 100 students is taken wrong so,
difference between 'm' and 'n' is 20, then what is marks of these students reduced from 75 to
the value of 'n'? 45. Now the average of all students in class
(a) Quantity I > Quantity II reduced by 24 marks. Find the value of B.
Quantity II. 25 boys or 40 girls can do a job in 25
(b) Quantity I < Quantity II
days. A contractor deploys 20 boys and 24 girls
(c) Quantity I ≥ Quantity II
for this work, but after every duration of 10 days
(d) Quantity I ≤ Quantity II
5 boys and 8 girls left the job. So work is
(e) Quantity I = Quantity II or relation can't be
completed in (d/18 + 15) days. Find the value of
established.
d.
8. Quantity I. K is a positive integer formed by A. Difference between B and d = 5
using digits 0, 1, 2, 3, 4, 5 using each digit at
B. B > d
most one. Find the number of possible values
C. B + d > 250
of K if K > 2000.
Quantity II. A bag contains 2C pink balls, C white (a) A and C only (b) A and B only
balls and B black balls. Probability of picking a (c) B and C only (d) A, B, and C
black ball from bag is 1/5, and probability of (e) None of these

Direction(Q10-13): The data given below shows the information of two types of articles sold by 5
sellers. The line graph given below shows the total number of articles (P & Q) sold by 5 sellers. The
column graph shows the selling price (in Rs.) of each article and average selling price of each article.
Read both graphs carefully and answer the following questions based on it

600
510
480
500 450

400 360
330

300

200

100

0
P Q R S T

5
DAY-21
70
60 60
60 58 57
54 54
52 52
50 48 48
46
44
39
40 36 36

30

20

10

0
P Q R S T

Selling price of article M Selling price of article N Average selling price of each article

Note: Average selling price of each article =


𝑠𝑒𝑙𝑙𝑖𝑛𝑔 𝑝𝑟𝑖𝑐𝑒 𝑜𝑓 𝑎𝑟𝑡𝑖𝑐𝑙𝑒 𝑀 ×𝑡𝑜𝑡𝑎𝑙 𝑛𝑜.𝑜𝑓 𝑎𝑟𝑡𝑖𝑐𝑙𝑒 𝑀+𝑠𝑒𝑙𝑙𝑖𝑛𝑔 𝑝𝑟𝑖𝑐𝑒 𝑜𝑓 𝑎𝑟𝑡𝑖𝑐𝑙𝑒 𝑁×𝑡𝑜𝑡𝑎𝑙 𝑛𝑜.𝑜𝑓 𝑎𝑟𝑡𝑖𝑐𝑙𝑒 𝑁
𝑡𝑜𝑡𝑎𝑙 𝑛𝑜.𝑜𝑓 𝑎𝑟𝑡𝑖𝑐𝑙𝑒 𝑀+𝑡𝑜𝑡𝑎𝑙 𝑛𝑜.𝑜𝑓 𝑎𝑟𝑡𝑖𝑐𝑙𝑒 𝑁

10. Number of article M sold by seller Q is (a) 22 (b) 26


approximately what percentage more/less (c) 29 (d) 33
than number of article N sold by seller S? (e) 24
(a) 56% (b) 71%
(c) 35% (d) 31% Direction(Q14-17): Read the information given
(e) 62% in paragraph carefully and answer the questions
given below.
11. Ratio of cost price of article M and N for seller
Aman and Naman are two farmers and they are
R is 5: 3. He marked the price of article M 50%
ploughing their filed. Field of Aman is rectangular
above cost price and that of article N above and he can plough at the rate of 150 m2/hour and
cost price. He offers discount of 20% on takes total 16 hours. Efficiency of Naman is
article M and 10% on article N. Find the total 13(1/3)% more than that of Aman and Naman takes
profit earned by him by selling all the articles. same time to plough his field as Aman takes. Field of
(a) Rs.4420 (b) Rs.3760 Naman is a equilateral triangle of side (P) meter.
(c) Rs.5150 (d) Rs.5840 Ratio of sides of field of Aman is 3: 2 and he runs
(e) Rs.3360 around his field with speed (Q) km/h and complete
12. A seller F sold article M at 20% above the one revolution in 14.4 seconds.
selling price of same article sold by seller R If ratio of efficiency of Aman and Naman remains
and he sold article N at less than the selling same for any work and time taken by Aman alone to
price of same article sold by seller T. By this, fill a tank is (R) hours. Aman and Naman together
he generated a revenue of Rs.12640. Find start filling the tank and work for 5 hours after that
total article sold by F, if difference between Aman left and Naman alone works for 10 more
hours. Now, Naman replaced by Aman who takes 12
article M and article N sold by him is 20, in
more hours to fill the remaining part of tank.
which article M sold are more.
Capacity of the tank is (S) litres and filling efficiency
(a) 250 (b) 280
of Naman is 1.57 litres/minutes. Water from the tank
(c) 240 (d) 320
is now poured into a large cylindrical vessel of base
(e) 220 radius 100 cm. After filling the whole amount of
13. Find the difference between average articles water from the tank into vessel height of water in the
M sold by all the sellers and average articles vessel becomes (T) cm.
N sold by all the sellers. (Use √3 = 1.7 and π = 3.14 and 1liter= 1000cm≥)

6
DAY-21
14. Two cars initially certain distance apart, they 18. M ml from container E and N ml from
meet after 18 minutes and 2.7 hours when container F is removed and poured in vessel
travelling in opposite direction and same S (empty) such that % of honey in vessel S
direction respectively. Speed of faster car is becomes (C – 5) %. When 132 ml of water is
(Q) km/h, then what is the initial distance added to vessel S then ratio of honey and
between those cars? water in vessel S becomes N: M. Find 5(M + N)
(a) 24 km (b) 27 km is how many times more than that of D?
(c) 30 km (d) 36 km (a) 10 (b) 9
(e) None of these (c) 12 (d) 7.5
(e) None of these
15. A truck can cover a certain distance in (R)
hours and a car can cover the same distance 19. Vessel F mixed with vessel S (mixture of
in 42.5 hours. If the speed of truck is 10 km/h honey and water, contains K% water) in ratio
more than the speed of car, then what is the of 3:5 respectively, such that resultant
speed of car? mixture contains X/3 ml mixture. Resultant
(a) 40 km/h (b) 30 km/h mixture mixed with X ml volume of vessel E
(c) 45 km/h (d) 50 km/h such that final mixture contains (W – 5) %
(e) None of these water. Find the concentration of Honey in
vessel S initially?
16. An amount of Rs.(S) is distributed among (a) 20% (b) 80%
three persons M, N and O is such a way that
(c) 75% (d) 60%
amount of M is Rs.370 less than the amount of
O and amount of O is Rs.202 less than the (e) Can’t be determined
amount of N, then what is the average of 20. From vessel E, 20% mixture is removed and
amount of M and O? replaced by equal quantity of water and then
(a) Rs.935 (b) Rs.624 again one third of the mixture is removed and
(c) Rs.813 (d) Rs.738 replaced by same quantity of water. Now
(e) None of these mixture in vessel E poured in vessel F such
that total quantity of mixture vessel F
17. Volume of a sphere and cone is same, if the becomes 1600 ml and concentration of water
base radius of cone is (T) cm and height 45 becomes 45%. Find the initial quantity of
cm, then what is the cost of paining the mixture in vessel E which is added to vessel
surface of sphere of cost of paining is F?
Rs.5/cm2? (Use π = 3.14) (a) 400 ml (b) 750 ml
(a) Rs.107120 (b) Rs.98270
(c) 800 ml (d) 1200 ml
(c) Rs.107230 (d) Rs.127170
(e) None of these
(e) None of these
21. A number of pipes of type-1 can fill a tank in z
hours. 3M pipes of type-2 can fill the same
Direction(Q18-20): Read the following
tank in 3/4 z hours. 6M pipes of type-3 can fill
information carefully and answer the questions the same tank in 3z hours. A number of type-
based on it. 1 pipes, A number of type-2 pipes, A number
Three containers P, Q, and R contains mixture of of type-3 pipes together can fill the tank in
honey and water. 40/3 hours.
If 2M type-1 pipes working (z-10) hours, 2M type-2
Container % of honey % of water pipes working (z-5) hours and 2M outlet pipes
E 75% 25% working t hours having efficiency same as that of
F 60% 40% type-3 pipe together fill the tank. Find the value
G 45% 55% of t?
Note: When E and G are mixed in 1:5 ratio (a) 120 hours (b) 130 hours
respectively then resultant mixture contains (11W/6 (c) 100 hours (d) 50 hours
– 70/3) % water. (e) 75 hours

7
DAY-21
𝒛
22. What is the value of n = 𝒛𝟏𝟎 Note: Books owned by library = Books issued by
(a) 225 (b) 400 library + Number of books available in library
(c) 250 (d) 525 24. If in another library T which has only two
(e) both 1 and 2 type of books English and Hindi, total number
23. (M+5) type-1 pipes, (M+10) type-2 pipes and of books is 25% less than that of P. Number of
12M type-3 pipes working 5 hours can fill a books not issued by library T is 60 less than
tank of same capacity given in the question. that of library S, if number of English books in
Find total time taken by them to fill the tank? the library T is 60 less than that of Hindi
In another case (2M+5) type-1 pipes, (10-M) books, then find the number of Hindi books
type-2 pipes and 2M type-3 pipes starts not issued by library T if it issued 40 English
together but type-1 pipes closed before 3 books?
hours to fill the tank and type-2 pipes closed
(a) 175 (b) 170
before 6 hours to fill the tank.
(a) 570/79 hours (b) 285/32 hours (c) 180 (d) 190
(c) 570/73 hours (d) 570/67 hours (e) 200
(e) 570/71 hours 25. In library R, there are only two types of
books- quantitative aptitude books and
Direction(Q24-25): Read the following
reasoning books, If out of these 20% of books
information carefully and answer the questions
based on it. removed and replaced by reasoning books,
Given data is about number of books contained in a then the no. of quantitative aptitude books
library, books are either owned or issued by library. and the no. of reasoning books becomes
Library Number of books Number of equal, then find the number of quantitative
owned by library books issued aptitude books in the library?
by library (a) 286 (b) 270
P 560 160 (c) 262 (d) 266
Q 460 200 (e) Can’t be determined
R 720 400
S 640 260

Direction (26-28): There are ten vessels G, H, I, J, K, M, N, O, P & Q. Bar shows graph total milk in G, H,
I, J & K as a percent of total milk in M, N, O, P & Q respectively. Table below shows ratio of total water
in G, H, I, J & K to total water in M, N, O, P & Q respectively. Read the data carefully and answer the
question.
140 130
125
120
IN PERCENTAGE

100
80 80
80 75

60

40

20

G H I J K

8
DAY-21

Vessels Ratio of water in vessel


G:M 7:5
H:N 1:2
I:O 3:5
J:P 3:4
K:Q 1:5

26. Milk in M is double of water. 60% of mixture (a) I and III only (b) II and III only
from G and 80% of mixture from M are taken (c) II only (d) I, III, and III
out, so that average cost of this resulting (e) None of these
mixture become 40 Rs/litre. and average cost
of milk in resulting mixture 50 Rs./litre If the 30. The quadratic equations d² – 6d + m = 0 and
difference between total cost of resulting d² - nd + 6 = 0 have one root in common and
mixture and total cost of resulting milk is the other roots of equations (i) and (ii) are
2000 Rs., then find the difference between integers in the ratio of 4 : 3 then, what is the
qmilk & water in resulting mixture (Cost of value of common root?
water also included)? (a) 1 (b) 2
(a) 48 liters (b) 56 liters
(c) 3 (d) 4
(c) 54 liters (d) 46 liters
(e) 5
(e) 84 liters
27. Milk in H is 120% more than that of water in 31. A, B and C started the business
that vessel and difference between water in N simultaneously after 10 months C left the
and H is 2000 liters. If water in P is 800 liters business and after another 6 months B left
less than milk in H and ratio of milk and water the business. Total profit at the end of two
in P is 16 : 9, then find milk in J (in liters)? years is E, out of which profit share of A is Rs.
(a) 8320 (b) 8120 D. Find the value of (E + 500).
(c) 8430 (d) 8350
Statement I. Profit share of B is (D + 2000) or (N
(e) 8310
+ 500) where N is the profit share of C.
28. Total mixture in I is 72% of total mixture in O.
Statement II. Profit share of C: (profit share of A
Mixtures from I & O are mixed in the ratio of
3 : 4 and from this resulting mixture, is and B together) = 15:28
replaced with same quantity of water. If again Statement III. Initial investment of A and B
10% of resulting mixture is replaced with together is same as investment of C.
same quantity of water, then find the ratio of (a) Only I and II together is sufficient to answer
milk to water in resulting mixture? the question
(a) 33 : 43 (b) 33 : 41 (b) Only I and III together is sufficient to answer
(c) 33 : 31 (d) 33 : 35 the question
(e) 33 : 37
(c) Either (I and II) together or either (I and III)
29. An examination consists of 24 questions. together is sufficient to answer the question.
Each correct answer carried 1 mark, each
(d) All three statement together is sufficient to
wrong carried (- 0.5) marks and each
unattempt question carried ( - 0.25) marks. answer the question
Score of Raman in that exam is 10 marks. (e) None of these
I. Minimum possible number of unattempted 32. There are three members in a family- Ram,
questions is 4. Rohini, and , Shyam and their present ages in
II. maximum possible number of wrong ratio of B, A and C years respectively
questions is 6.
(integers). Shyam is son of Ram, and Rohini is
III. Difference between minimum and maximum
possible number of correct questions is 2. wife of Ram and B > A > C. Find the ratio of the
ages of Ram and Rohini?

9
DAY-21
Statement I. Average age of Rohini and Ram (a) only II
after six years is 40 years, ratio of present age of (b) only I
Rohini to that of age of Shyam after 13 years is (c) Either I or II
32:19. (d) Both statements together are sufficient.
Statement II. Age of Ram before 6 years is 2.5 (e) None of these
times of age of Shyam after six years. Average age
of family after 2 years is 80/3 years.

Direction(Q33-36): Person P entered in three businesses (M, N, and O) with different partners. In M,
partner of P is Q. In N, partners of P are R and T. In O, partner of P is S. The following bar graph shows
the ratio of initial investment of P and Q respectively in business M, that of P, R and T respectively in
business N and that of P and S respectively in business O.
6
5
4
3
2
1
0
Business Business NBusiness O
M
P Q R S T
Durations of business M, N and O are 2 years, 3 years and 4 years respectively.
In business M, P and Q entered together at the start. After 8 months Q withdraws all his investment and after
16 months from start of business, P withdraws 30% of his initial investment. Q came back at the starting of
21st month from the start of business with 50% more than his initial investment. At the end of two years of
business, difference between the profit shares of P and Q is Rs.10120.
In business N, P and R entered together at the start. After 6 months P and R withdraw Rs.2000 and Rs.4000
respectively and T joined P and R at the same time. After 10 months since T joined, T doubled his investment
and at the starting of 25th month from the start of business, P invested Rs.5000 more and at the starting of
31st month from the start of business, R invested Rs.20000 more. At the end of 3 years of business, the profit
share of T is Rs.6250.
In business O, profit ratio between P and S at the end of 4 years of business is 6: 5 respectively and the
difference between the profit shares of P and S is Rs.5260.

33. Find the difference between the total profit in 34. The initial investment of P in business O is
business M and N together and total profit in Rs.6000 less than the initial investment of P
business O. in businesses M and N together. Total initial
(a) Rs.7920 (b) Rs.8090 investment of P in all three businesses
(c) Rs.8270 (d) Rs.7860 together is Rs.36000. Which of the following
(e) Rs.8150 can be the possible initial investment of P in
business M and N respectively?

10
DAY-21
(i) 12000, 9000 (a) If the data in statement I is not sufficient to
(ii) 15000, 6000 satisfy the question.
(iii) 9000, 15000 (b) If the data in statement II is not sufficient to
(a) Only (i) and (ii) (b) Only (ii) and (iii) satisfy the question.
(c) Only (i) (d) Only (i) and (iii) (c) If the data in statement III is not sufficient to
(e) None of these satisfy the question.
35. A began the business O initially and joined by (d) Any statement I, II, or III can satisfy the
S after 't' months. Find the value of 't' (in question.
months).( Assume that there is no change in (e) None of these
investment and no one left the business once
joined) Directions(Q38-40): Three series I, II, and III
following a certain pattern are given below. In
(a) 10 (b) 6 the series, one of the numbers is missing in the
(c) 8 (d) 5 numbers in series I, in series II, and in series III
(e) 9 are represented by 'A', ''B', and 'C' respectively.
Series I: 29, 58, 174, 696, ______.
36. Profit share of P is what percent less than the
Series II: 49, 99, 199, 399, _____, 1599
profit share of R in business N? Series III: _____, 359, 503, 334, 530, 305
(a) 25% (b) 30% Consider the given number series and find the wrong
(c) 45% (d) 40% term.
(e) 36% C, (B + 161), 6C, (3A + 2C + 121), (C x 120)
1. (3A + 2C + 121)
The following question is accompanied by three
2. (C x 120)
statements (I), (II), and (III). You have to 3. C
determine which statements(s) is/are 4. (B + 161)
sufficient/necessary to answer the questions. 5. 6C
37. P and Q together started a business by 38. Find the minimum number that can be either
added or subtracted from B, to make the
investing the amount in ratio 3: 4. After four resultant number a perfect square number.
months, R joins with the contribution of the (a) 18 (b) 16
amount which is m% more than that of P. (c) 15 (d) 14
After 9 months, Q withdraws his (e) 10
contribution. At the end of 2nd year, the 40. Which of the following statements is/are
difference between the profits earned by Q's correct about the value of (A + C²) - B?
(a) Is perfectly divisible by 61
and R's is n times the difference between the
(b) Is more than (B)2
profits earned by P and R (c) Sum of digits is a composite number
I. m = 60%, n = 5/2 (d) Both 1 and 2
II. m = 30%, n = 7 (e) Both 2 and 3
III. m = 50%, n = 3

11
DAY-21

Solutions
1. Ram can complete his work by working "m" hrs is 7.
in "a" days or by working "m-4" hrs in III: et2 – 17t + 3s = 0
"a+5"days e * 72 - 17 * 7 + 21 = 0
m×a = (m * 4) × (a + 5) -> 5m - 4a = 20 49e - 119 + 21 = 0
Possible values of m and a -> ((8, 5), (12, 10)) 49e = 98
Shyam completes his work by working "n" hrs e=2
in "b" days or by working "n-4" hrs in "b+5" III: 2t2 – 17t + 21 = 0
days 2. If both the roots of a quadratic equation become
n×b = (n - 4) × (b + 5) -> 5n - 4b = 20 equal which means the equation will become a
Possible value of n and ((8, 5), (12, 10)) perfect square.
Now work completed by Ram & Shyam together (√2t)2 - 2 * √2t * (17/2√2) + (17/2√2)2 + 21 -
in 1 day will be (17/2√2)2 = 0
(When a = 5 and b = 5 ) (√2t - 17/2√2)2 + 21 - (289/8) = 0
required days-2.5 days (√2t - 17/2√2)2 - (121/8) = 0
(When a = 5 and b = 10 ) Hence, 121/8 must be added to equation III so
required days-10/3 days that both of its root become equal.
(When a = 10 and b = 10 ) 3. d = 37
required days =5 days e=2
Correct option is 2. r=5
s=7
Soln.(2-3): 37 = 5 * 7 + 2
One root of the equation I = 2/5 d=r*s+e
I: 5m2 - dm + 14 = 0 d = rs + e
5 * (2/5)2 - d * (2/5) + 14 = 0
4. Let two boats P and Q covers 240 km and 340
(4/5) - (2d/5) + (70/5) = 0
km downstream distance in a hours and b hours
4 – 2d + 70 = 0
respectively. The respective ratio of
2d = 74
downstream of boat P to the speed of boat Q in
d = 37
still water is c respectively. Stream is flowing
5m2 – 37m + 14 = 0
with speed d km/hr. Then,
5m2 – 35m – 2m + 14 = 0
Speed of boat Q in still water = 24 km/hr
5m (m - 7) - 2 (m - 7) = 0
Difference between upstream speed of both
(m - 7) (5m - 2) = 0
boats = difference between downstream speed
m = 7 and 2/5
of both boats = difference between speed of
Since, one non-integer root in equations I and II
both boats in still water
is common. Which means one root of equation II
Then, speed of boat P in still water = 24 - 2 = 22
is 2/5 and another root is 1.
km/hr
(n - 1) (n - 2/5) = rn2 - sn + 2 = 0
Speed of stream = d = 22 * 100/(100 + 120) =
(y - 1) (5y - 2) = py2 - sn + 2
10 km/hr => d = 10
5n2 – 2n – 5n + 2 = rn2 - sn + 2
Then, upstream speed of boat P = 22 - 10 = 12
5n2 – 7n + 2 = rn2 - sn + 2
km/hr
After comparing:
Downstream speed of boat P = 22 + 10 = 32
r=5
km/hr
s=7
Upstream speed of boat Q = 24 - 10 = 14 km/hr
Since, one integer root in equations I and III is
Downstream speed of boat Q = 24 + 10 = 34
common. Which means one root of equation III

12
DAY-21
km/hr day = 'R' units
Then, a = 240/32 => a = 7.5 Total time taken to finish the work when Sam
b = 340/34 => b = 10 and David are working = 37/(5 + R) = 4(14/15)
c = 32: 24 => c = 4: 3 = 74/15
5. Interest earned from scheme A = (5E + 1200) x 15 = 10 + 2R
(25/16 – 1) = (45E/16 + 675) R = 2.5
Amount received from scheme B = (45E/16 + Time taken by R alone to finish the work =
675) x 36/25 = (81E/20 + 972) 100/2.5 = 40 days.
Simple interest received from scheme D = Quantity II:
(81E/20 + 972 - E) x 12% x 5 = 2E + 80 Since the difference between 'm' and 'n' is 20.
(61E/20 + 972) x 3/5 = 2E + 80 m ~ n = 20
17E/100 = 2516/5 Case 1: When (m - n) = 20
Value of E = 2960 m = (n + 20)
Required sum = (61P/20 + 972) = Rs. 10000 According to question-
[100 - m]% of [100 - n]% of 12000 = 2880
6. Let rate of inflow is a liters/hour and rate of
[100 - (n + 20)]% of [100 - B]% of 12000 = 2880
outflow is b liters/hour.
[80 - n]% of [100 - n]% of 12000 = 2880
Now,
(80 - B)(100 - n) = 2400
M + 8a = 24 x 8 x b…………… (1)
n2 – 180n + 5600 = 0
Also,
(n - 140)(n - 40) = 0
M + 16a = 20 x 16 x b………... (2)
n = 40
On dividing both equations, we get
m = 40+20 = 60
(M + 8a) / (M + 16a) = 24 x 8 x b / 20 x 16 x b
n can't be more than 100 because person can't
M = 4a
spend more than what he has with him.
Form equation (1)
Case 2: When (n - m) = 20
4a + 8a = 24 x 8 x b
n = (m + 20)
Value of a = 16b
According to question-
Now,
[100 - m]% of [100 - n]% of 12000 = 2880
(4a + 4 x a) / (4 x b) = 2N + 8
[100 - m]% of [100 - (m + 20)]% of 12000 =
8 x 16b / 4b = 2N + 8
2880
2N + 8 = 32
[100 - m]% of [80 - m]% of 12000 = 2880
Value of N = 12
(80 - m)(100 - m) = 2400
7. Quantity I: m2 – 180m + 5600 = 0
Ratio of efficiency of Sam and Tom = (1/4): (m - 140)(m - 40) = 0
(1/5) = 5: 4 m = 40
Let time taken by Sam and Tom is 4x and 5x n = m + 20 = 60
respectively. m can't be more than 100 because person can't
(1/4x) + (1/5x) = (9/100) spend more than what he has with him.
x=5 So, the possible value of n is 40 or 60.
Time taken by Sam alone = 4x = 20 days Quantity I < Quantity II
Time taken by Tom alone = 5x = 25 days
8. Quantity I.
Let total work = LCM of 20 days and 25 days =
K > 2000
100 units
Number is = 4 digit or 5 digit or 6 digits
Work done by Sam and Tom together in first 7
When number is 4 digits
days = 7 * (5 + 4) = 63 units
First place filled by = 2, 3, 4, 5
Remaining work = 100 - 63 - 37 units
Next places filled in = 5 x 4 x 3 ways
Let amount of work done by David alone in 1

13
DAY-21
Total numbers = 4 x (5 x 4 x 3) = 240 B + d < 250
When number is five digits So, this statement is wrong.
Required value = 5 x (5 x 4 x 3 x 2) = 600
When number is six digits Soln.(10-13):
Required number = 5 x (5 x 4 x 3 x 2 x 1) = 600 Seller P:
Required value of K = 600 + 600 + 240 = 1440 Let article M Sold by P be a.
Quantity II 60a + (330 - a) ×54 = 58 ×330
According to question 60a + 17820 - 54a = 19140
B / (C + 2C + B) = 1/5 6a = 1320
5B = 3C + B a = 220
B/C = 3/4 [3a, 4a] So, article M and N Sold by seller P are 220 and
Now, 110 respectively.
(4a/15a) x (8a / (15a – 1)) = 16/105 Seller Q:
Value of a = 1 Let article M Sold by seller Q be b.
So, value of A = 15 x 1 = 15 54b + 39× (450 - b) = 46 × 450
Required value = 100 x 15 – 20 x 3 = 1440 54b + 17550 - 39b = 20700
So, Quantity I = Quantity II 15b = 3150
Hence answer is option C b = 210
9. Quantity I. So, article M and N Sold by seller Q are 210 and
According to question, 240 respectively.
[B x A – 100 x (75 – 45)]/B = A – 24 Seller R:
BA – 3000 = BA – 24B Let article M Sold by seller R be c.
Value of B = 125 60c + 36× (510 - c) = 52×510
Quantity II. 60c + 18360 - 36c = 2652r
25B x 25 = 40G x 25 24c = 8160
B/G = 8/5 c = 340
Total work = 25 x 8 x 25 = 5000 units So, article M and N Sold by seller R are 340 and
Amount of work done in first 10 days = (20 x 8 + 170 respectively.
24 x 5) x 10 = 2800 units Type here to search
Amount of work done in next 10 days = (15 x 8 Seller D:
+ 16 x 5) x 10 = 2000 Let article P Sold by seller D be d.
Amount of work done left after 20 days =5000 - 48d + 57× (360 - d) = 52 × 360
(2000 + 2800) = 200 units 48d + 20520 - 57d = 18720
Time taken to complete the remaining work = 9d = 1800
200 / (10 x 8 + 8 x 5) = 5/3 days d = 200
d/18 + 15 = 20 + 5/3 So, article M and N Sold by seller S are 200 and
d/18 = 20/3 160 respectively.
d = 120 Seller T:
A. Difference between B and d = 5 Let article P Sold by seller T be e.
Required difference = 125 – 120 = 5 36e + (480 - e) × 48 = 44 × 480
This statement is true. 36e + 23040 - 48e = 21120
B. B > d 12e = 1920
125 > 120 e = 160
This statement is true. So, article M and N Sold be seller T are 160 and
C. B + d > 250 320 respectively.
B + d = 125 + 120 = 245 10. Required percentage = (210 - 160)/160 * 100

14
DAY-21
= 125/4% P2 = 6400
= 31.25% ≅ 31% P = 80 m
11. Let cost price of article M and N be 50x and 30x Total distance to be covered by Aman around
respectively his field = 2 * (60 + 40) = 200 meters
150 According to question-
Marked price of article M = 50x × 100 = 75x
(200/1000) = Q * (14.4/3600)
80
Selling price of article P = 75𝑥 × = 60x Q = (0.2/0.004)
100
60x = 60, x= 1 Q = 50 km/h
So, total profit by selling article M = 10 × 340 = Ratio of efficiency of Aman to Naman = 150: 170
𝑅𝑠. 3400 = 15: 17
Cost price of article N = Rs.30 Let time taken by Naman alone to fill the tank =
Total profit by selling article N = 6 × 170 = Rs. R * (15/17) = 15C/17
1020 Total time for which Aman works = 5 + 12 = 17
Total profit = 3400 + 1020 = Rs.4420 hours
12. Let article N sold by seller F be x Total time for which Naman works = 5 + 10 = 15
120 5 hours
60 × 100 × (𝑥 + 20) + 48 × 6 × 𝑥 = 12640
According to question-
72x + 1440 + 40x = 12640 [17/R] + [(15 * 17)/15R] = 1
112x = 11200 34/R = 1
x = 100 R = 34 hours
So, total article sold = x + x + 20 Time taken by Naman alone to fill the tank =
100+ 100 + 20 = 220 15R/17 = 30 hours
13. Required difference Filling efficiency of Naman = 1.57 litres/minutes
220+210+340+200+160 110+240+170+160+320 Capacity of the tank = T = (1.57 * 60) * 30
= 5
− 5
1130 1000 S = 2826 litres
= −
5 5 Amount of water in the tank = 2826 litres =
= 226 – 200 = 26 2826000 cm3
Amount of water in the tank = Amount of water
Soln(14-17): in the vessel
Aman can plough his field at the rate of 150 Volume of tank = Volume of vessel up to height
m2/hour and takes total 16 hours. of T cm.
Total area of rectangular field = 150 * 16 = 2400 2826000 = π(100)2T
m2 2826000 = 3.14 * 10000 * T
Ploughing rate of Naman = 113(1/3)% of 150 = T = 2826000/31400
170 m2/hour T = 90 cm
Total area of field of Naman which is in the Q = 50
shape of equilateral triangle = 170 * 16 = 2720
14. Let the speed of slower car = x km/h
m2
Effective speed when travelling in same
Let the sides of field of Aman are 3x and 2x
direction = (50 - x) km/h
respectively.
Effective speed when travelling in opposite
3x * 2x = 2400
direction = (50 + x) km/h
6x2 = 2400
Let the distance between them = D km
x = 20
According to question-
Sides of field of Aman = 60 m and 40 m
D/(50 - x) = 2.7 ....... (1)
respectively.
D/(50 + x) = (18/60) = 0.3 ..... (2)
(√3/4)P2 = 2720
From (1) and (2)-
(1.7/4)P2 = 2720

15
DAY-21
(50 + x)/(50 - x) = 9 Value of V = 30
450 + 9x = 50 - x % Of water in F = V + 10 = 30 + 10 = 40%
10x = 400 % Of milk in F (B) = 60%
x = 40 km/h % Of milk in G = 60 – 15 = 45%
From equation (1)- % Of water in G = 55%
D/(50 - 40) = 2.7 55 = 2W – V + 5
D = 27 km 50 + 30 = 2W
15. R = 34 Value of W = 40
Let the distance = D km So, % of water in E = 40 – 15 = 25%
Let the speed of car and truck is 'S' and 'S + 10' % Of milk in E (C) = 75%
respectively. Container % of honey % of water
According to question- E C% (W - 15) %
D/(S + 10) = 34 ...... (1) F D% (V + 10) %
D/S = 42.5 .... (2) G (D - 15)% (2W – V + 5) %
From (1) and (2)- According to question,
42.5S = 34S + 340 18. When E and F are mixed, honey % in S becomes
8.5S = 340 = (C – 5) = 70%
S = 40 Now,
Speed of car = 40 km/h 75% 60%
16. S = 2826 70%
Let amount of N = Q 10% 5%
Amount of O = (Q - 202) That means E and F are mixed in = 2:1
Amount of M = (Q - 202 - 370) = (Q - 572) Ratio of honey and water in S, now = 7:3
According to question- 132 ml water is added, then ratio of honey and
(Q - 572) + Q + (Q - 202) = 2826 water becomes = 1:2
3Q - 774 = 2826 Now,
Q = 1200 7a / (3a + 132) = 1/2
Required average = [(Q - 572) + (Q - 202)]/2 = 14a = 3a + 132
Rs.813 11a = 132
17. T = 90 cm Value of a = 12
So, initial quantity of mixture S = 12 x 10 = 120
Let the radius of sphere = R cm
Volume of sphere = Volume of cone ml
Value of M = 2 / (2+1) x 120 = 80 ml
(4/3)πR3 = (1/3)π(90)2 * 45
4R3 = (90)2 * 45 Value of N = 1 / (2+1) x 120 = 40 ml
Value of 5(M + N) = 5 x (80 + 40) = 600 ml
R3 = (45 * 45 * 45)
Value of D = 60
R = 45 cm
Surface area of sphere = 4π(R)2 = 25434 m2 Required ratio = 600:60 = 10:1
So, 5(M + N) is 9 times more than D
Cost of painting = 25434 * 5 = Rs.127170
Soln(17-20): When E and G are mixed in ratio 19. According to question,
of 1:5 respectively, then resultant mixture When F and S mixed in 3:5, then concentration
contains (11W/6 – 70/3) % water. of water in resultant mixture = [(40 x 3 + K x
Now, 5)/8] %
(W – 15) x 1 + 5 x (2W – V + 5) = 6 x (11W/6 – Now,
70/3) (120 + 5K)/8 x 1 + 25 x 3 = (Z – 5) x 4
W – 15 + 10W – 5V + 25 = 11W – 140 120 + 5K + 600 = 140 x 8
11W – 5V + 10 = 11W – 140 5K = 400

16
DAY-21
Value of K = 80 x = 180
So, concentration of honey in vessel S = 100 – 80 Number of Hindi books in library T = 180+60
= 20% =240
Hence answer is option A Number of English books issued by library T =
40
20. Concentration of honey in vessel E after
Therefore, number of Hindi books issued = 100
certain removal and replacements = 75% x 4/5
– 40 =60
x 2/3 = 40%
Number of Hindi books not issued by library T =
Now,
240 – 60
= 180
285
t= 32
ℎ𝑜𝑢𝑟𝑠
25. Total books in library R = 720
After replacement total reasoning books =
That means mixture in vessel E (after removal)
quantitative aptitude books = 360
and mixture in vessel F is in ratio = 5:15 = 1:3
20% of books removed from the library, i.e 20%
So, quantity of mixture in vessel E after removal
of 720 = 144
= 1/4 x 1600 = 400 ml
These 144 books are replaced with reasoning
21. Let type-1 pipes efficiency be t1. books. The ratio of quantitative aptitude and
Type-2 pipes efficiency by t2. reasoning books is not given, there number of
Type-3 pipes efficiency by t3. quantitative aptitude books in the library can’t
3𝑧
M× 𝑡1 × 𝑦 = 3𝑀 × 𝑡2 × = 6𝑀 × 𝑡3 × 3𝑧 be determined.
4
,𝑡1: 𝑡2: 𝑡3 = 18: 8: 1
Let 𝑡1: 𝑡2: 𝑡3 = 18𝑘: 8𝑘: 1𝑘 Soln(26-28):
40
M× (18𝑘 + 8𝑘 + 1𝑘) × = 𝑀 × 18𝑘 × 𝑧 26. Let milk and water in M be 100x and 50x
3
y = 20 respectively
80
20 Milk in G = 100x × 100 = 80x
22. 20 = 400
10
7
Water in G = 50x × 5 = 70x
23. (M+5)× 18𝑘 × 5 + (𝑀 + 10) × 8𝑘 × 5 +
60
12𝑀 × 𝑘 × 5 = 𝑀 × 18𝑘 × 20 Total milk in resulting mixture = 80x × 100 +
9M + 45 + 4M + 40 + 6M = 36M 80
100𝑥 × = 128𝑥
100
M=5 80
Total water in resulting mixture = 50x × 100 +
Let tank fill in t hours
60
15× 18𝑘 × (𝑡 − 3) + 5 × 8𝑘 × (𝑡 − 6) + 70𝑥 × 100 = 82𝑥
10 × 𝑘 × 𝑡 = 5 × 18𝑘 × 20 Total resulting mixture = 128x + 82x = 210x
32t = 285 ATQ
24. Total number of books in library T = 75% of 560 210x × 40 − 128𝑥 × 50 = 2000
= 420 x=1
Number of books not issued by library T = (640- Required difference = 128 – 82 = 46 litres
260)-60 = 320 2000
27. Water in H = 2−1
× 1 = 2000𝑙𝑖𝑡𝑟𝑒𝑠
Number of books issued by library T = 420 – 2000
320 = 100 Milk in H = 100 × 220 = 4400𝑙𝑖𝑡𝑟𝑒𝑠
Let number of English books in library T =x Water in P = 4400 – 800 = 3600 litres
Therefore, number of Hindi books in library T = 3600
Milk in P = 9
× 16 =6400L
x+60; 6400
Milk in J = 100 × 130 =8320L
(x+ x+60) = 420

17
DAY-21
28. Let total milk I & O be 4a & 5a respectively Required difference = 13 – 14 = 1
And water in I & O be 3b & 5b respectively Only I and II is correct.
(4𝑎+3𝑏) 18 Hence answer is option E
ATQ, =
(5𝑎+5𝑏) 25
20a + 15b = 18a + 18b 30. Equation (i) : d2 – 6d + m = 0
a: b = 3:2 Let the root of this equation be = a and b
12 2 Sum of roots = 6
Part of milk in I = 18 = 3
⇒a+b=6
15 3
Part of milk in O = 25 = 5 Product of roots = ab = m
Let part of milk in resulting mixture = n Equation (ii) d2 - nd + 6 = 0
𝑛−
3
3 Let the common root be 'a'.
By allegation we know = 2 5
=4
−𝑛
3
And, the other root of this equation = c
22
n = 35 Sum of roots = a + c = n
Product of roots = ac =6
let total resulting mixture = 35y
Other roots 'b' and 'c' are integers and in the
Part of milk in resulting mixture = 22y
ratio of 4 : 3.
Part of water in resulting mixture = 35y – 22y =
Let b = 4k and c = 3k
13y
2
Therefore, a + 4k = 6
After replacing 16 3 % of resulting mixture with ⇒ a = 6 - 4k ...(a)
same amount of water Given : ac = 6
50 1 55𝑦
Remaining milk = 22y – 22y × × 100 = 3 ⇒ (6 - 4k) × (3k) = 6
3
50 1 ⇒ 18k - 12k2 = 6
Remaining water = 13y - 13y × × + 35y
3 100 ⇒ 2k2 - 3k + 1 = 0
50 1 50𝑦
× 3
× 100 = 3
⇒ 2k2 - 2k - k + 1 = 0
Again replaced 10% of resulting mixture with ⇒ 2k × (k - 1) - 1 × (k - 1) = 0
amount of water ⇒ (2k - 1) (k - 1) =0
55𝑦 55𝑦 10 ⇒ k = 0.5 or 1
Milk in final mixture = 3
− ( 3 × 100) = 16.5𝑦
Roots are integers therefore, k = 1
50𝑦 50𝑦 10
Water in final mixture = −( × )+ b = 4 and c = 3
3 3 100
35𝑦 ×
10
= 18.5y Common root = a = 6 - 4k = 6 - 4 = 2.
100
16.5𝑦 31. Ratio of time period of investment of A, B, and C
Required ratio = = 33: 37
18.5𝑦 = 24:16:10 = 12:8:5
29. According to question, From statement I, we don’t have any
C + W + U = 24……………. (1) information about investment, so this statement
C – 1/2 x W – 1/4 x U = 10……………... (2) alone is not sufficient.
Form (1) and (2) From II.
5C – W = 64 Let investment of A, B, and C = A, B, and C
If W = 1, C = 13, and U = 10 Now,
If W = 6, C = 14, U = 4 Ratio of profit share = A x 12: B x 8: C x 5 =
No other values are possible 12A:8B: 5C
I. Minimum possible number of un attempted Now,
question is 4 (True) 5C / (12A + 8B) = 15/28
II. maximum possible number of wrong 9A + 6B = 7C
questions is 6 (True) From III.
III. Difference between minimum and maximum Let investment of A, B, and C = A, B, and C
possible number of correct questions is 2 A+B=C
(False) On Combining II and III, we get

18
DAY-21
9A + 6B = 7C Present age of Ram = 68 – 32 = 36 years.
A+B=C If age of Shyam after 13 years = 38 years
So, Present age of Shyam = 38 – 13 = 25 years
9A + 6B = 7A + 7C Present age of Rohini = 64 years
A/B = 1/2 Present age of Ram = 68 – 64 = 4 years (not
So, Ratio of investment of A, B, and C = 1:2:3 possible)
Ratio of profit = 12:16:15 So, age of Rohini = 32 years
This combination is not sufficient to answer the Age of Ram = 36 years
question. Required ratio = 36:32 = 9:8
On combining I and II, This statement alone is sufficient to answer the
Profit of A = D question.
Profit of B = D + 2000 Statement II.
Profit of C = D + 2000 - 500 = D + 1500 B – 6 = 5/2 x (C + 6)
Now, 2B - 12 = 5C + 30
(D + 1500) / (2D + 2000) = 15/28 2B – 5C = 42…………. (1)
Let profit is in thousands for calculation Also,
convenient B + A + C + 6 = 3 x 80/3
14D + 21 = 15D + 15 B + A + C = 74…………... (2)
D=6 We need B and A, so eliminate C,
So, combined profit = 6 + 6 + 2 + 6 + 1.5 = Rs. On solving both equations, we get
21500 7B + 5A = 412
This combination is sufficient to answer the A = (412 – 7B)/5
question. 7B must contains unit digit 2, when 7 multiply 6
On combining (I + III) gives 2-unit digit
A+B=C Also, 2B > 42
Ratio of profit share of A, B, and C = 12A: 8B: B > 21
(5A + 5B) So, if B = 26, A = 46 (not possible)
Difference between profit share of A and B / If B = 36, A = 32
Difference between profit share of B and C = Then C = 6
(8B – 12A) / (8B – 5A – 5B) = 2000/500 If B = 46, A = 18
2B – 3A = 3B – 5A Then B = 10
A/B = 1/2 So, we can’t find the exact ages.
So, ratio of profit share of A, B and C = 12:16:15 So, this statement alone is not sufficient to
Total profit = 43/4 x 2000 = 21500 answer the question.
This combination is sufficient to answer the So Only I is sufficient to answer the question.
question
So, Either I and II together or Either I and III Soln(33-36):
together In business M,
32. From I. Initial investment of P = Rs.3a
B + A + 12 = 40 x 2 Initial investment of Q = Rs.2a
B + A = 68………. (1) Duration of business M = 2 years.
Also, ratio of age of Rohini to Shyam after 13 Profit ratio of P and Q = [3a * 16 + (21a/10) * 8]:
years is 32:19 (2a * 8 + 3a * 4) = [48a + (84a/5)]: 28a = 324:
If age of Shyam after 13 years = 19 140 = 81: 35
Present age of Shyam = 19 – 13 = 6 years Difference between the profit shares of P and Q
So, at that time age of Rohini = 32 years = Rs.10120
So,

19
DAY-21
46 units = Rs.10120 Profit share of T = Rs.6250
1 unit = Rs.220 So, 25 units = 6250
81 units = Rs.220 * 81 = Rs.17820 1 unit = Rs.250
35 units = Rs.220 * 35 = Rs.7700 27 units = Rs.250 * 27 = Rs.6750
Profit share of P = Rs.17820 45 units = Rs.250 * 45 = Rs.11250
Profit share of Q = Rs.7700 Profit share of A = Rs.6750
Total profit in business M after 2 years = Profit share of C = Rs.11250
Rs.17820 + 7700 = Rs.25520 Total profit in business Y = Rs.24250
In business N, In business O,
Initial investment of P = Rs.3b Profit ratio of P and S = 6: 5
Initial investment of R = Rs.5b Difference between the profit share of P and S =
Initial investment of T = Rs.2b Rs.5260
Profit ratio of P, R, and T = (3b * 6 + (3b - 2000) So, 1 unit = Rs.5260
* 18 + (3b - 2000 + 5000) * 12): (5b * 6 + (5b - 11 units = Rs.5260 * 11 = Rs.57860
4000) * 24 + (5b - 4000 + 20000) * 6): (2b * 10 6 units = Rs.5260 * 6 = Rs.31560
+ 4b * 20) 5 units = Rs.5260 * 5 = Rs.26300
= (18b + 54b - 36000 + 36b + 36000): (30b + Profit share of P = Rs.31560
120b - 96000 + 30b + 96000): (20b + 80b) Profit share of S = Rs.26300
= 108b: 180b: 100b = 27: 45: 25 Total profit in business O = Rs.57860
Profit share of Profit share of Profit share of Profit share of Profit share of Total profit
Business P (in Rs.) Q (in Rs.) R in Rs.) S (in Rs.) T (in Rs.) (in Rs.)

M 17820 7700 25520

N 6750 11250 6250 24250

O 31560 26300 57860

33. Required difference = 57860 - (25520 + 24250) Investment of P in business N = Rs.15000


= 57860 - 49770 = Rs.8090 Then investment of P in business O = 9000 +
15000 - 6000 = Rs.18000
34. Checking from options,
Sum = 9000 + 15000 + 18000 = Rs.42000
From (i),
Hence option (iii) is not possible.
Investment of P in business M = Rs.12000
Investment of P in business N = Rs.9000 35. Let, initial amount invested by P = Rs. X
Then investment of P in business O = 12000 + So, initial amount invested by S = Rs. X
9000 - 6000 = Rs.15000 Duration of business O = 4 years = 48 months
Sum = 12000 + 9000 + 15000 = Rs.36000 Profit ratio of P and S = 6: 5
Hence option (i) is possible. M * 48 : M * (48 - t) = 6: 5
From (ii), 6 units = Rs.48M
Investment of P in business M = Rs.15000 1 unit = Rs.8M
Investment of P in business N = Rs.6000 5 units = Rs.40M
Then investment of P in business O = 15000 + So,
6000 - 6000 = Rs.15000 M * (48 - t) = 40M
Sum = 15000 + 6000 + 15000 = Rs.36000 48 - t = 40
Hence option (ii) is possible. t = 8 months.
From (iii), 36. Required percentage = (11250 - 6750) *
Investment of P in business M = Rs.9000 100/11250 = 450000/11250 = 40%

20
DAY-21
37. the ratio of A and B investment = 3: 4 ⇒ 799 + 800 = 1599
Let R investment after 4 months be y B = 799
After 9 months Q withdraws his amount = 4 × 9x Series III: _____, 359, 503, 334, 530, 305
After 2 years profit ratio = 3x:× 24: 4x × 9: y × The logic followed here is as follows:
20 ⇒ 480 - 121 = 359
⇒ P: Q: R = 72x: 36x: 20y ⇒ 18x: 9x: 5y ⇒ 359 + 144 = 503
Difference between Q and R = |9x - 5y| ⇒ 503 - 169 = 334
Difference between P and R = |18x - 5y| ⇒ 334 + 196 = 530
Ratio of difference = |9x−5y|/|18x−5y| ⇒ 530 - 225 = 305
Substitute the option I C = 480
R is 160% of P = 3x × 160% = 4.8x = y 38. C, (B + 161), 6C, (3A + 2C + 121), (C x 120)
5y = 4.8x × 5 = 24x So the series is:
|9x−5y|/18x−5y|= |9x−24x||18x−24x|= 480, 960, 2880, 11521, 57600
15x/6x= 5/2 The logic followed here is:
So it could satisfy ⇒ 480 x 2 = 960
Substitute the option II = 30%, 2 ⇒ 960 x 3 = 2880
R is 130% of P = 3x × 130% = 3.9x = y ⇒ 2880 x 4 = 11520
5y = 3.9x × 5 = 19.5x ⇒ 11520 x 5 = 57600
|9x−5y|/|18x−5y|| = |9x−19.5x|/|18x−19.5x| = 1 Hence, the wrong term is 11521'
0.5x/1.5x=7/1 Hence, the correct answer is option 1.
So it could satisfy
39. C = 480
Substitute the option III = 50%, 3
B = 799
R is 150% of P = 3x × 150% = 4.5x = y
Let's check option 1:
5y = 4.5x × 5 = 22.5x
Adding 18 and subtracting 18 from 799, we get
|9x−5y|/|18x−5yI = |9x−22.5x|/|18x−22.5x| = 1
817 and 781 respectively
3.5x/4.5x= 3/1
Let's check option 2:
So it could satisfy
Adding 16 and subtracting 16 from 799, we get
But here given options combination of two
815 and 783 respectively
options
Let's check option 3:
Soln.(38-40):
Adding 15 and subtracting 15 from 799, we get
Series I: 29, 58, 174, 696, ______
814 and 784 respectively
The logic followed here is as follows:
(28)2 = 784
⇒ 29 x 1 = 29
⇒ 29 x 2 = 58 40. Value of (A + C2) - B
⇒ 58 x 3 = 174 = [(3480 + (480)2] - (799)
⇒ 174 x 4 = 696 = (3480 + 230400) - 799
⇒ 696 x 5 = 3480 = 233880 - 799
A = 3480 = 233081
Series II: 49, 99, 199, 399, _____, 1599 = 233081/61 = 3821
The logic followed here is as follows: Is perfectly divisible by 61 (True).
⇒ 49 + 50 = 99
⇒ 99 + 100 = 199
⇒ 199 + 200 = 399
⇒ 399 + 400 = 799

***

21
DAY-21

To enroll Rank File click here : Use Code Y179S to get maximum Discount

To enroll Bank Mahapack click here : Use Code Y179 to get maximum Discount

22

You might also like